Groups of Order 30: Unique Sylow-5 Subgroup?

  • Thread starter Thread starter math8
  • Start date Start date
  • Tags Tags
    Groups
Click For Summary
Any group of order 30 has a normal Sylow-5 subgroup, which is unique due to the structure of its Sylow subgroups. The analysis shows that if there are six Sylow-5 subgroups, it leads to a contradiction regarding the number of elements of various orders in the group. A cyclic normal subgroup of order 15 exists within any group of this order, which guarantees the uniqueness of the Sylow-5 subgroup. Consequently, the Sylow-5 subgroup is normal in the larger group. This confirms that every group of order 30 indeed has a unique normal Sylow-5 subgroup.
math8
Messages
143
Reaction score
0
Is it true that any group of order 30 has a normal (hence unique) Sylow-5 subgroup?

I know that that the only possibilities for n(5) are 1 or 6.

Now suppose there are 6 sylow 5 subgroups in G. This would yield
(5-1)6=24 distinct elements of order 5 in G. Now there is only 30-24=6
elements left in G and one of these is the identity. This means that there
must be 1 sylow 3 subgroup in G which has 2 distinct elements of order 2
now there is only 4 elements left in G one of them being the identity
so there must be 3 sylow 2 subgroups of G in this case each having 1 distinct
element of order 2

I don't see where is the contradiction here.

The only thing I know is that there must be a cyclic normal subgroup of order 15 in G.
 
Last edited:
Physics news on Phys.org
Oh I think I got it.

Every group G of order 30 has a normal cyclic subgroup of order 15 (I can prove this).
Let's call it H.
Now, consider Syl_5(H).
n_5=1, hence if P lies in Syl_5(H), then P is the unique normal subgroup of H of order 5. Hence P char H. Since H is normal in G, it follows P is normal in G.

Now |P|=5 and |G|=(2^2)*3*5. So P is a normal Sylow-5 subgroup of G.
 
Question: A clock's minute hand has length 4 and its hour hand has length 3. What is the distance between the tips at the moment when it is increasing most rapidly?(Putnam Exam Question) Answer: Making assumption that both the hands moves at constant angular velocities, the answer is ## \sqrt{7} .## But don't you think this assumption is somewhat doubtful and wrong?

Similar threads

  • · Replies 6 ·
Replies
6
Views
2K
  • · Replies 1 ·
Replies
1
Views
2K
  • · Replies 9 ·
Replies
9
Views
2K
  • · Replies 5 ·
Replies
5
Views
3K
  • · Replies 1 ·
Replies
1
Views
2K
Replies
5
Views
2K
  • · Replies 3 ·
Replies
3
Views
2K
  • · Replies 4 ·
Replies
4
Views
6K
  • · Replies 9 ·
Replies
9
Views
2K
  • · Replies 1 ·
Replies
1
Views
5K